2023 AMC 8 Problems/Problem 11

Revision as of 21:17, 24 January 2023 by Princerohatgi (talk | contribs) (Written Solution)

Written Solution

Since the answers are so far apart, we just need to compute the # of figures the answer contains. So by approximating all the values for the hourly rate we have $\frac{300,000,000}{5 * 30 * 30} \approx \frac{300,000,000}{5000} = 60,000$ which is $\boxed{\text{(C)}60,000}$

~apex304, SohumUttamchandani, wuwang2002, TaeKim, Cxrupptedpat

Animated Video Solution

https://youtu.be/hwR2VM9tHJ0

~Star League (https://starleague.us)